Matemática, perguntado por starsdance20125412, 6 meses atrás

Prove por indução que:

Anexos:

Soluções para a tarefa

Respondido por klebersilf2020
1

Resposta:

Provado, eu acho.

Explicação passo-a-passo:

Lá em cima... nas três fotos.

Qualquer equívoco, só avisar.

Espero ter ajudado!!!

Anexos:
Respondido por Zecol
1

Vamos inicialmente provar que 1^2+2^2+\cdots+(n-1)^2<\frac{n^3}{3},\;\forall n\geq 2. Partindo do pressuposto de que essa relação é válida para um número natural k, ou seja, 1^2+2^2+\cdots+(k-1)^2<\frac{k^3}{3} é verdade, vamos provar que ela é válida para k+1:

1^2+2^2+\cdots+(k-1)^2+k^2<\frac{(k+1)^3}{3}

1^2+2^2+\cdots+(k-1)^2+k^2<\frac{k^3+3k^2+3k+1}{3}

1^2+2^2+\cdots+(k-1)^2+k^2<\frac{k^3}{3}+\frac{3k+1}{3}+k^2

1^2+2^2+\cdots+(k-1)^2+k^2-k^2<\frac{k^3}{3}+\frac{3k+1}{3}

1^2+2^2+\cdots+(k-1)^2<\frac{k^3}{3}+\frac{3k+1}{3}

De fato, se 1^2+2^2+\cdots+(k-1)^2<\frac{k^3}{3}, então 1^2+2^2+\cdots+(k-1)^2 também é menor que a soma de \frac{k^3}{3} por qualquer número não negativo, como é o caso de \frac{k^3}{3}+\frac{3k+1}{3}.

Para k=2, a inequação é verdadeira pois 1^2<\frac{2^3}{3} e, como a inequação também é verdadeira para k+1, ela é válida para qualquer número natural maior que 1.

Vamos agora provar que \frac{n^3}{3}<1+2+\cdots+n^2,\;\forall n\geq 2. Partindo do pressuposto de que essa relação é válida para um número natural k, ou seja, \frac{k^3}{3}<1+2+\cdots+k^2 é verdade, vamos provar que ela é válida para k+1:

\frac{(k+1)^3}{3}<1+2+\cdots+k^2+(k+1)^2

\frac{k^3+3k^2+3k+1}{3}<1+2+\cdots+k^2+k^2+2k+1

\frac{k^3}{3}+\frac{1}{3}+k^2+k<1+2+\cdots+2k^2+2k+1

\frac{k^3}{3}+\frac{1}{3}<1+2+\cdots+2k^2+2k+1-k^2-k

\frac{k^3}{3}+\frac{1}{3}<1+2+\cdots+k^2+k+1

\frac{k^3}{3}<1+2+\cdots+k^2+k+1-\frac{1}{3}

\frac{k^3}{3}<1+2+\cdots+k^2+k+\frac{2}{3}

Assim como no caso anterior, se \frac{k^3}{3} é menor que 1^2+2^2+\cdots+n^2, então ele também é menor que 1^2+2^2+\cdots+n^2 somado a qualquer número não negativo, como é o caso de k+\frac{2}{3}.

Para k=2, a inequação é verdadeira pois \frac{2^3}{3}<1^2+2^2 e, como a inequação também é verdadeira para k+1, ela é válida para qualquer número natural maior que 1.

Com isso, prova-se que 1^2+2^2+\cdots+(n-1)^2<\frac{n^3}{3}<1^2+2^2+\cdots+n^2,\;\forall n\geq 2.

Perguntas interessantes